NCLEX Hurst Adult/Maternity/Psy/Priority/Child/Fundamental Questions

Réussis tes devoirs et examens dès maintenant avec Quizwiz!

The emergency room nurse is assessing a client with an eye injury that occurred while chopping wood. The client states the chain saw caused a log to splinter, sending slivers of wood into the right eye. While waiting for the eye specialist, the nurse discusses future safety precautions for such an activity. What safety precautions are most important for the nurse to include in client teaching? Select all that apply 1. Wear heavy gloves. 2. Stand with feet together. 3. Use steel-toed boots. 4. Wear unbreakable googles. 5. Use ear covers and plugs. 6. Wear loose-fitting clothing.

1, 4 and 5. CORRECT. When engaging in a potentially risky activity, precautions should be taken even if the activity has been completed multiple times before. Functional body parts, such as hands, fingers and toes, are particularly vulnerable to injury. Heavy duty work gloves made of leather or suede along with protective eye googles should be worn even before turning on any machines. Ears should also be protected with regulation ear phones or ear plugs because of equipment noise levels. 2. INCORRECT. Any activity involving equipment poses a safety risk, no matter how often an individual completes that action. A client should have both feet firmly planted on a flat surface, approximately shoulder-width apart, with weight distributed evenly over the hips. Standing with feet together distributes body weight unevenly, increasing the risk for injury. 3. INCORRECT. While sturdy leather boots provide protection for the feet, it is not necessary to have steel-toed boots. However, the client should never wear sneakers, sandals or other light-weight, non-protective foot wear when using any type of machinery or equipment. 6. INCORRECT. Loose fitting clothing could easily become caught in equipment, yanking the body in towards sharp blades and other moving parts. A client needs snug fitting clothing to cover exposed extremities to prevent even minor injuries.

A client with Hepatitis C has returned from surgery with a total laryngectomy. The nurse knows that what personal protective equipment is necessary when providing trach care? (Select All That Apply). 1. Face mask 2. Shoe covers 3. N-95 mask 4. Goggles 5. Gloves 6. Gown

1, 4, 5 and 6. CORRECT: The client has had a total laryngectomy which will initially produce large amounts of thick, bloody mucus. Hepatitis C is transmitted through blood and body fluids. During trach care, the nurse needs to be protected by specific personal protective equipment (PPE's). For this procedure, the nurse should utilize gown, gloves, goggles and face mask. 2. INCORRECT: Tracheostomy care is completed in close proximity to the client. Splattering of blood and body fluids on the floor is unlikely, so shoe covers are unnecessary. 3. INCORRECT: The N-95 face mask is a specially fitted mask used by nurses when providing care for clients with active tuberculosis. It is not necessary for a client with Hepatitis C.

A client has been admitted with advanced Cirrhosis. The nurse's assessment reveals an abdominal girth increase of 5 inches (12.7 cm) and a weight increase of 6 lbs.(2.71 kg) since yesterday's measurements. What further assessment findings would the nurse expect? Select all that apply 1. Hypotension 2. Cool extremities 3. Bradycardia 4. CVP readng of 8 mm/Hg 5. Radial pulses 4+/4+

1. & 2. Correct: These are signs and symptoms of FVD due to 3rd spacing and shock is what you are afraid of. 3. Incorrect: We would expect the heart rate to increase in FVD in an effort to move what little volume you have left through the system. 4. Incorrect: This is a high CVP, and with FVD you would expect it to be low. 5. Incorrect: Pulses are evaluated on a 4 point scale, so 4 would be a bounding pulse which would indicate fluid volume excess.

The nurse plans to teach a client how to manage the use of a behind the ear hearing aid. What teaching strategies should the nurse include? Select all that apply 1. Hairspray should not be used while wearing the hearing aid. 2. A whistling sound when the hearing aid is inserted indicates proper placement. 3. Submerse hearing aid in cool water daily to clean. 4. Illustrate where damage commonly occurs on a hearing aid. 5. Batteries last 6 months with daily wearing of 10-12 hours.

1. & 4. Correct: The residual from the hair spray causes the hearing aid to become oily and greasy. The client should routinely inspect the hearing aid for damage, especially where damage is more likely: ear mold, earphone, dials, cord, and connection plugs.2. Incorrect: A whistling sound indicates incorrect ear mold insertion, improper fit of aid, and buildup of earwax or fluid.3. Incorrect: Do not submerse hearing aid in water, as it will damage the device.5. Incorrect: Batteries last 1 week with daily wearing of 10-12 hours.

While completing the nutritional history of a client admitted with pernicious anemia, the nurse determines that the client follows a strict vegan diet. What education should the nurse provide to the client? Select all that apply 1. Vitamin B12, a nutrient needed to prevent pernicious anemia, is found in some foods like meat, fish, eggs, and milk. 2. In order to increase intake of vitamin B12, your diet must contain beef or chicken liver at least once per week. 3. In addition to eating plants, you should eat dairy products and eggs in order to prevent pernicious anemia. 4. Vegetables high in protein include cabbage, carrots and squash. 5. Pernicious anemia occurs when the body produces red blood cells that are larger than normal and result in a lower than normal red blood cell count.

1. & 5. Correct: Pernicious anemia is a type of vitamin B12 anemia. The body needs vitamin B12 to make red blood cells. You get this vitamin from eating foods such as meat, poultry, shellfish, eggs, and dairy products. 2. Incorrect: The client does not have to eat meat or dairy products in order to obtain vitamin B-12. Supplements can be taken and the client can eat vegetables that are considered to be high in protein. 3. Incorrect: A strict vegan will not eat dairy products or eggs. 4. Incorrect: For a vegetable to qualify as a low-protein source, it must contain 4g or less of protein. Green vegetables, such as lettuce, cabbage, bell pepper and asparagus provide only 1 to 2g of protein per serving. Orange vegetables, including carrots, sweet potatoes and squash also contain only 1 to 2 g.

When assessing for the development of an infection following the application of a plaster cast to the leg, the nurse should teach the client to observe for the presence of which sign of infection? 1. Hot spots 2. Cold toes 3. Warm toes 4. Paresthesia

1. Correct: Hot spots is the best answer. Redness and increased warmth are indicators of localized infection. If the cast covers the extremity, redness cannot be visualized, but the client can feel more warmth (a "hot spot") in an area becoming infected. 2. Incorrect: "Cold toes" is a neurovascular check, not an indication of infection. 3. Incorrect: "Warm toes" is a neurovascular check, not an indication of infection. 4. Incorrect: Paresthesia is a neurovascular check, not an indication of infection.

An elderly, bed-bound client receiving G-tube feeding at home is transported to the emergency department after onset of behavioral changes and hallucinations. Which nursing action is priority while diagnostic testing is underway? 1. Initiate seizure precautions 2. Discontinue G-tube feeding 3. Administer oxygen 4. Obtain blood work for troponin level

1. Correct: Feeding tube clients tend to get dehydrated, especially clients on bed rest, because bed rest induces diuresis! If the client is already having neurological signs, a grand mal seizure may be next! Better take seizure precautions while awaiting the serum sodium results. 2. Incorrect: You may do this; however, seizure precautions will take priority. 3. Incorrect: The priority here is seizure precautions. 4. Incorrect: This data should lead to the suspicion of dehydration and hypernatremia, not suspected MI, which would be the reason a troponin level would be obtained.

A client is admitted to the hospital with acute exacerbation of COPD following an upper respiratory infection. His daughter found him at home, confused and in respiratory distress, a day after he developed a cold. He was placed on 4 L/min of oxygen via nasal cannula, but oxygen saturation remains at 89%. Based on this assessment, the nurse suspects that the client has developed which acid base imbalance? 1. Respiratory acidosis 2. Respiratory alkalosis 3. Metabolic acidosis 4. Metabolic alkalosis

1. Correct: Look at all the hints in this stem: COPD, upper respiratory infection, respiratory distress, confused, oxygen saturation of 89%. This client is having lung problems. So you should be able to identify the acid base imbalance as a respiratory problem, right? Yes. Why is it acidosis? Poor gas exchange! Respiratory failure, COPD, and muscular weakness can lead to respiratory acidosis. So you would expect the pH to be < 7.35, and the pCO2 to be > 45. The HCO3 would be normal. 2. Incorrect: Not alkalosis. You would expect respiratory alkalosis with a client who is hyperventilating, such as the hysterical client. The client in this question would be hypoventilating and having poor gas exchange. 3. Incorrect: Not a metabolic related acid/base imbalance. Metabolic problems do not start with a respiratory problem. Metabolic acidosis is seen with diabetic ketoacidosis or starvation. 4. Incorrect: This is not a metabolic problem but a respiratory problem. Metabolic alkalosis may be seen with prolonged vomiting and hypokalemia.

The nurse is caring for a postoperative client. The client asks the nurse the purpose of anti-embolic stockings. What is the nurse's best response? 1. Promotes the return of venous blood to the heart and assists in preventing blood clots. 2. Stabilizes any clots to prevent embolization. 3. To increase the blood pressure in the venous system in the legs to promote perfusion. 4. Promotes lymphatic drainage to prevent swelling and arterial congestion.

1. Correct: The anti-embolic stockings promote return of venous blood to the heart and assist in preventing the stasis of blood that can lead to blood clots. 2. Incorrect: The purpose of the anti-embolism stockings is to promote venous return and prevent blood stasis which can result in blood clot formation. Anti-embolitic stockings will not stabilize existing blood clots. 3. Incorrect: Anti-embolism stockings are used to increase venous return. They are not used to increase blood pressure or perfusion to the legs. 4. Incorrect: Compression garments, not anti-embolitic stockings, are used by persons with lymphedema to reduce edema by promoting the flow of lymph fluid out of the affected limb. Anti-embolitic stockings are to help with venous return and preventing stasis of blood and blood clots.

A client with an automated internal cardiac defibrillator (AICD) was successfully defibrillated. The telemetry technician shouts out that the client was in ventricular fibrillation (VF). What should the nurse do first? 1. Go to the client to assess for signs and symptoms of decreased cardiac output. 2. Call the primary healthcare provider to report that the client had an episode of VF so medication adjustments can be made. 3. Notify the "on call" person in the cath lab to re-charge the ICD in the event that the client has a recurrence. 4. Document the incident on the code report form and follow up regularly.

1. Correct: The client comes first. Check to see how they are doing by completing a head to toe cardiac output assessment. make sure to include LOC, vital signs, skin and urinary output assessment. 2. Incorrect: Do not call before you assess the client who may be unconscious if the arrhythmia has decreased their cardiac output. 3. Incorrect: This is not needed because there is a battery that keeps it charged, so that they don't have to re-charge after each shock. 4. Incorrect: Documentation is not appropriate until the client has been assessed first.

A client has been admitted to the orthopedic floor following application of a long leg cast for a fractured femur. What nursing action takes priority? 1. Perform neurovascular checks of the extremities. 2. Cover the edge of the cast near the groin area. 3. Instruct client not to insert anything into cast. 4. Use palms of hands to lift and position the cast.

1. Correct: The most vital aspect of care for clients with a fracture and/or cast is frequent neurovascular checks. Circulation can quickly become compromised secondary to edema from the injury or application of the cast, leading to permanent nerve and tissue damage. Neurovascular checks are performed every two hours for the first 24 hours, or more often per hospital protocols, and both extremities must be compared when looking for problems. 2. Incorrect: While this is a vital action by the nurse, it is not the initial priority. Because this client has a long leg cast for a fractured femur, there is the potential for urine to contaminate the cast close to the groin. That would impair the integrity of the cast, or potentially cause an infection. The nurse definitely needs to cover the upper edges of the cast near the groin with water proof material, but there is another action to complete first. 3. Incorrect: Clients must always be instructed on self care or equipment function as part of the recovery process. Proper cast care following discharge is essential and, in particular, the importance of not placing anything down inside the cast. Clients tend to complain of itching skin beneath a cast and may put baby powder, corn starch or other objects inside the cast to scratch. All these can cause serious complications, and the nurse must provide specific teaching to prevent such problems. However, those instructions are not the most immediate priority for the nurse at this time. 4. Incorrect: Casting material can take up to 24 hours to dry hard enough to protect the client's injury. In the meantime, careful handling of the cast when positioning the client is crucial. The nurse is aware that the cast must be lifted using only the palms of the hands to prevent indentations which could injure the client's skin beneath the casting material. These instructions must also be relayed to any personnel providing care to the client; however, this is not the first priority.

A client presents to the after-hours clinic with reports of pain that occurs with walking but generally subsides with rest. The nurse's assessment reveals coolness and decreased pulses in lower extremities bilaterally. What condition would the nurse recognize these symptoms being most indicative of? 1. Chronic Arterial Insufficiency 2. Chronic Venous Insufficiency 3. Chronic Unstable Angina 4. Chronic Coronary Artery Disease

1. Correct: These symptoms are indicative of arterial insufficiency as there is pain with walking that is relieved by rest. This pain is known as intermittent claudication. In addition, the pulses are decreased or may be absent with arterial insufficiency and the extremities are cool to touch. Other s/s include: paleness of extremity when elevated or possible redness when lowered, loss of hair on affected extremity, and thick nails. 2. Incorrect: Venous insufficiency is not characterized by pain with walking. Pulses are generally normal and color is generally normal with the exception of the brown pigmentation that may be noted (especially around the ankles). 3. Incorrect: The description in the stem is evident of peripheral arterial insufficiency and is not descriptive of decreased coronary artery perfusion. No reports of chest pain were noted. 4. Incorrect: The description is evident of peripheral arterial insufficiency and is not descriptive of decreased coronary artery perfusion. The symptoms listed in the stem are indicative of a peripheral artery problem.

A client admitted with somnolence has a history of chronic bronchitis and heart failure. Vital signs on admit are T 101.8ºF (38.8ºC), HR 106, R 26/shallow, BP 90/58. ABGs are pH 7.2, PCO2 75, HCO3 26. The nurse determines that this client has which acid/base imbalance? 1. Respiratory acidosis 2. Respiratory alkalosis 3. Metabolic acidosis 4. Metabolic alkalosis

1. Correct: This client has a respiratory problem. Respiratory failure, COPD, and muscular weakness can lead to respiratory acidosis. Signs & symptoms: hypoventilation, sensorium changes, somnolence, semicomatose to comatose state. pH < 7.35, pCO2 > 45, HCO3 normal. 2. Incorrect: This is not alkalosis since the pH is 7.2 showing acidosis. 3. Incorrect: Not a metabolic related acid/base imbalance, because the HCO​2 is 26 and within the normal range. 4. Incorrect: Not a metabolic related acid/base imbalance, because the HCO​2​ is 26 and within the normal range.

Twelve hours post coronary artery bypass surgery (CABG), the nurse notes the client's level of consciousness has decreased from alert to somnolent. BP 88/50, HR 130 and thready, resp 32, urinary output (UOP) has dropped from 100 mL one hour earlier to 20 mL this hour. What would be the nurse's first action? 1. Administer 100% oxygen per mask. 2. Lower the head of the bed. 3. Give furosemide STAT. 4. Re-check the BP in the other arm.

1. Correct: This client has developed signs of cardiogenic shock, one of the complications post CABG. Cardiac output is decreased, so the client needs more oxygen for the circulating blood volume. 2. Incorrect: Lowering the HOB will not help in cardiogenic shock but will actually make it harder for the heart to pump. 3. Incorrect: Poor kidney perfusion is the reason for the decreased UOP. The kidneys are trying to conserve what little volume the body has to maintain vital organ perfusion as long as possible. 4. Incorrect: Rechecking the BP will not help the problem. With the other symptoms, this BP is most likely accurate. This would only delay treatment and would not fix the problem.

A nurse educator is teaching first responders about disaster management, and provided the following scenario: A small community has experienced a severe tornado that hit a shopping mall and caused extreme damage and suspected mass casualties and injuries. First responders arrive on the scene. The nurse educator recognizes education has been successful when the first responders identify which action as priority?

1. Triage victims and tag according to injury. 2. Assess the immediate area for electrical wires on the ground and in vicinity of victims. 3. Activate the community emergency response team. 4. Begin attending to injuries as they are encountered. 3

What should the nurse include in the teaching plan for a client receiving external beam radiation? 1. Small marks will be placed on the skin to mark the treatment area. 2. Lotion may be used around the treatment area to decrease dryness. 3. The radiation therapist can see, hear, and talk with you at all times during treatment. 4. Stay away from babies for 24 hours. 5. You will have to hold your breath during radiation treatment.

1., & 3. Correct: Small ink marks or small tattoos will be placed on the skin to mark the treatment area. Do not remove the marks. The radiation therapist can see, hear, and talk to the client at all times during treatment. Relieve anxiety by letting client know he/she is not alone. 2. Incorrect: Do not put lotion, powder or deodorant near or on treatment area. 4. Incorrect: Client is not radioactive and will not radiate others. The client can safely be around other people, babies, and children. 5. Incorrect: The client will need to stay very still so radiation goes to the exact same place each time, but can breathe as always and does not have to hold breath.

The homecare nurse is instructing the family of a client recently diagnosed with Parkinson's disease about potential neurologic changes. During the discussion, what signs should the nurse include? Select all that apply 1. Unsteady gait 2. Muscle rigidity 3. Hyperactive reflexes 4. Bradykinesia (slowed movements) 5. Expressive aphasia

1., 2 & 4. Correct: Parkinson's disease is a debilitating, progressive neurological disorder of unknown cause. The most classic symptoms include unsteady gait secondary to increasing muscle rigidity and bradykinesia, plus difficulty with purposeful movement. These symptoms worsen over time and are often accompanied by tremors in the extremities at rest. 3. Incorrect: Reflexes in clients with Parkinson's disease become progressively slowed, not hyperactive. Because this disorder affects the midbrain, and ultimately the connection of the basal ganglia, deep tendon reflexes decrease over the course of the disease. Hyperactive reflexes are associated with other neurologic disorders such as multiple sclerosis. 5. Incorrect: Expressive aphasia is associated with brain trauma or cerebral vascular accident (CVA) and prevents the client from verbalizing appropriate or desired terminology. In Parkinson's disease, the client's speech volume becomes too low and very monotone. Also, because of facial muscle rigidity, there is great difficulty articulating words enough to be clearly understood.

A client who has been trying to lose weight reports to the nurse that it is just easier to stop by the fast food restaurant on the way home from work than to go home and prepare a meal. Which interventions could help the client stay on track? Select all that apply 1. Suggest that the client eat yogurt and a piece of fruit upon returning home. 2. Suggest that the client order low fat options at the restaurants. 3. Encourage the client to pack a healthy snack to eat on the way home from work. 4. Inform the client that fast food restaurants do not have healthy food options. 5. Suggest that the client alter her route home from work in the evenings to avoid the fast food restaurants.

1., 2. & 3. Correct: The client is describing lack of convenience, a barrier to making better choices. The client can consume yogurt and fruit on the way home and still be making a good choice for dinner. Accessibility of healthier items will help the client stay on track. Availability of healthy foods will help the client stay on her food plan. 4. Incorrect: There are healthier choices currently at most fast food restaurants. Clients should be encouraged to choose from those.5. Incorrect: While this may help the client stay on track, it may make a healthy choice inaccessible. The client is more likely to make healthy choices when they are accessible, available, and affordable.

A home health nurse is interpreting Mantoux skin test results of clients who received the test 48 hours ago. Which clients have a positive tuberculin skin test reaction? Select all that apply 1. HIV+ client with an induration of 6 millimeters. 2. Client who immigrated from Haiti 6 months ago who has an induration of 10 millimeters. 3. Client working at a nursing home with an induration of 8 millimeters. 4. 3 year old client with an induration of 12 millimeters. 5. Healthy client with no known TB exposure who has an induration of 5 millimeters.

1., 2. & 4. Correct: HIV infected clients are considered to have a (+) TB skin test with an induration of 5 millimeters or more. An induration of 10 millimeters or more is considered positive in recent immigrants (less than five years) from high-prevalence countries such as Haiti, and in children less than 4 years of age. 3. Incorrect: An induration of 10 millimeters or more is considered positive for residents and employees of high-risk congregate settings. 5. Incorrect: An induration of 15 millimeters or more is considered positive in any person with no known risk factors for TB.

A client's central venous pressure (CVP) reading has changed significantly from the last hourly reading. Which data would the nurse assess that reflect changes in the CVP reading? Select all that apply 1. Heart sounds 2. Skin turgor 3. Temperature 4. Nail bed color 5. EKG rhythm 6. Urinary output

1., 2. & 6. Correct: The CVP reading reflects the client's fluid volume status. If the CVP is elevated, indicating FVE, then the nurse is likely to hear S3 sounds when auscultating the heart sounds. The client's skin turgor and urine output would reflect the client's fluid volume status. 3. Incorrect: The CVP reading reflects the client's fluid volume status. The client's temperature would not reflect the client's fluid volume status. 4. Incorrect: The CVP reading reflects the client's fluid volume status. The nail bed color would not reflect the client's fluid volume status. 5. Incorrect: The CVP reading reflects the client's fluid volume status. The EKG rhythm would not reflect the client's fluid volume status.

A palliative care client is suffering from persistent diarrhea. What foods should the nurse suggest? Select all that apply 1. Applesauce 2. Rice 3. Bananas 4. Tea 5. Yogurt

1., 2., & 3. Correct: The BRAT diet is recommended for clients with persistent diarrhea. This diet consists of bananas, rice, applesauce, and toast. Rice and potatoes help to reduce diarrhea. Bananas will help replace potassium. Once the diarrhea subsides, the client can add easily digestible foods like eggs. 4. Incorrect: Avoid coffee and tea because caffeine containing beverages may have a laxative effect. Caffeine is a stimulant and will increase the peristalsis even more. 5. Incorrect: Dairy products may make the diarrhea worse. Avoid these until the diarrhea subsides.

What interventions can an occupational health nurse discuss with a client in an effort to improve lateral epicondylitis (tennis elbow) pain? You answered this question Incorrectly 1. Avoid activities that make the pain worse. 2. An oral, nonsteroidal, anti-inflammatory drug may be prescribed. 3. Immediately start stretching and exercising the muscle and tendon. 4. If pain persists, a cortisone injection into the inflamed area may be recommended. 5. Apply ice for 45 minutes, six times a day.

1., 2., & 4. Correct: General activities that make the pain worse should be avoided or at least cut back. While continued activity in the presence of mild discomfort is not harmful, activities that cause severe pain will only prolong the necessary recovery time and should be avoided. Oral, nonsteroidal, anti-inflammatory drugs are very helpful in controlling the pain and inflammation of tennis elbow. The medicine is taken daily for at least four to six weeks when treating severe cases. For less severe cases, these medicines may be taken only when needed. Cortisone injections are considered when the other measures have not worked and the pain is severe. The cortisone is injected into the area of the inflamed tendons in order to decrease the inflammation. 3. Incorrect: Stretching and exercising of the involved muscle and tendon unit is one of the mainstays of treatment for this condition once pain and inflammation have subsided, but not during the acute phase. A gentle stretching program is started through a range of motion at the elbow and wrist. This is combined with a program of muscle strengthening. 5. Incorrect: It is recommended to apply ice to the area two to three times a day, for 20 to 30 minutes each time.

A client who has diabetes calls the nurse hot-line reporting shakiness, nervousness, and palpitations. Which questions would yield information that would help the nurse decide that this is a hypoglycemic episode? Select all that apply 1. What have you eaten today and at what times? 2. Are you using insulin as a treatment of diabetes, and if so, what kind? 3. Do you feel hungry? 4. Do you have access to a glucose monitor to check your current glucose level? 5. Does your skin feel hot and dry?

1., 2., 3. & 4. Correct. This question will give the nurse information about how much time has elapsed since the last meal and will indicate the amount of protein and carbohydrates consumed at the last meal. Even a minor delay in meal times may result in hypoglycemia. Insulin type will give the nurse information about duration of action and peak time. Hunger is a symptom of hypoglycemia.If the client has a glucose monitor, an accurate reading would give the nurse valuable information about how much food the client should consume now.5. Incorrect. Hot and dry skin is not an indicator of hypoglycemia and would not help the nurse determine if the client is experiencing a hypoglycemic episode. Cool, clammy skin is a symptom of hypoglycemia.

A home health nurse is planning home safety education for a client and spouse. Which actions should be included to promote fire safety in the home setting? Select all that apply 1. A fire extinguisher should be kept on each level of the home. 2. Keep matches and lighters away from children by storing them in a locked cabinet. 3. Install carbon monoxide smoke alarms, and test them monthly. 4. You may leave Christmas lights lit all night as long as the tree is artificial. 5. Have a planned route of exit and a place where all family members will meet.

1., 2., 3. & 5. Correct: A fire extinguisher should be placed on each level of the home, near an exit, but out of reach of children. Keeping matches and lighters away from children by storing them in a locked cabinet can prevent fire-related deaths. Carbon monoxide smoke alarms will alarm for smoke as well as carbon monoxide, which is an odorless gas than can kill quickly. Alarms should be tested every month and repaired or replaced immediately if malfunction occurs. A plan facilitates exit from the building, and a place to meet helps identify that all family is out of the building. 4. Incorrect: Lit Christmas lights should be turned off when no one is home and when people go to bed for the night. It does not matter whether the tree is real or artificial.

When providing care for a client with a chest drainage unit (CDU) set at 20 cm. of suction, which nursing actions are correct? Select all that apply 1. Maintain chest drainage system below the client's chest during transport. 2. Apply tape to the tubing connection sites. 3. Add sterile water to suction control chamber to achieve 20 cm. 4. Assess respiratory effort every shift. 5. Ensure that tubing is not kinked or looped.

1., 2., 3. & 5. Correct: Never raise the drainage system above the level of the client's chest. All connection sites should be tightly secured. If the water level drops below the prescribed suction, more saline must be added. Tubing must not be kinked or looped. 4. Incorrect: Respiratory effort for a client with a CDU must be assessed more often than every shift. This assessment should be done at least every 2 hours.

A client is preparing to be discharged after a total hip replacement. Which client statement would indicate teaching has been successful regarding prevention of hip prosthesis dislocation? Select all that apply 1. "I should not cross my affected leg over my other leg." 2. "I should not bend at the waist more than 90 degrees." 3. "While lying in bed, I should not turn my affected leg inward." 4. "It is necessary to keep my knees together at all times." 5. "When I sleep, I should keep a pillow between my legs."

1., 2., 3. & 5. Correct: These are appropriate actions to prevent hip prosthesis dislocation. Until the hip prosthesis stabilizes it is necessary to follow these instructions for proper positioning to avoid dislocation. 4. Incorrect: The knees should be kept apart to prevent dislocation.

The nurse is discussing frostbite prevention with a group of teenagers who participate in cold weather activities. What risk factors for developing frostbite will the nurse include? Select all that apply 1. Alcohol use 2. Dehydration 3. Diabetes 4. Exhaustion 5. Low level altitude

1., 2., 3., & 4. Correct: Risk factors for developing frostbite include alcohol and drug abuse, dehydration, medical conditions such as diabetes or any condition that results in poor blood flow to the extremities, fatigue and exhaustion. 5. Incorrect: Being at a high altitude reduces the oxygen supply to extremities and places the person at increased risk for developing frostbite.

What signs of cannula displacement should the nurse monitor for at an arterial line insertion site? Select all that apply 1. Swelling 2. Fluid leakage 3. Blanching 4. Poor arterial waveform 5. Pyrexia 6. Purulent drainage

1., 2., 3., & 4. Correct: These are signs of cannula displacement. Observe for signs of cannula displacement into the tissues which will be swelling, bleeding, lack of a normal arterial waveform, fluid leakage, blanching, and pain or discomfort. 5. Incorrect. This is a sign of infection rather than cannula displacement. Signs of infection include pain, redness, purulent drainage, and fever. 6. Incorrect. This is a sign of infection rather than cannula displacement.Signs of infection include pain, redness, purulent drainage, and fever.

Which food items, if chosen by a client diagnosed with diverticulosis, would indicate to the nurse that the client understands the prescribed diet? Select all that apply 1. Avocados 2. Acorn squash 3. Applesauce 4. Lima beans 5. Raspberries 6. Cottage cheese

1., 2., 4., & 5. Correct: High fiber foods include raw fruits, legumes, vegetables, whole breads, and cereals. Avocados have 10.5 grams of fiber per cup. Acorn squash has 9 grams of fiber per cup. Lima beans 13.2 grams of fiber per cup. Raspberries have 8 grams of fiber per cup. 3. Incorrect: Raw fruits have more fiber than cooked or processed fruits. A raw apple would provide more fiber than applesauce. 6. Incorrect: Milk and foods made from milk: such as yogurt, pudding, ice cream, cheeses, cottage cheese and sour cream are low fiber.

A nurse is caring for a client diagnosed with the ebola virus who is experiencing vomiting and diarrhea. What personal protective equipment should be worn by the nurse while providing care to this client? Select all that apply 1. Single use impermeable gown 2. Powered Air Purifying Respirator (PAPR) or N95 respirator 3. One pair of sterile gloves 4. Single use boot covers 5. Single use apron

1., 2., 4., & 5. Correct: The nurse should wear a single use (disposable) impermeable gown OR a single use impermeable coverall. Either a PAPR or a disposable, NIOSH-certified N95 respirator should be worn to reduce the risk of contamination in the case of an emergency situation where a potentially aerosol-generating procedure would be performed. The PAPR reduces the risk of self-contamination while providing client care, but the N95 respirator is less bulky. If the N95 respirator is selected for use, nurses should be extremely careful to make sure that they do not accidentally touch their faces under the face shield during client care. Disposable boot covers should be worn and should extend to at least mid-calf. Some agencies may add the single use shoe covers over the boot covers to reduce the risk of contaminating the underlying shoes. If clients with Ebola are vomiting or have diarrhea, a single use (disposable) apron should be worn over the gown to cover the torso to mid-calf. This will provide additional protection to reduce the risk of contaminating the gown (or coveralls) by the infectious body fluids and also provides a way to rapidly remove a soiled outer layer if contamination occurs on the apron. 3. Incorrect: Sterile gloves are not required, but two pairs, instead of one pair, of gloves should be worn so that a contaminated outer glove can be safely removed when providing client care or safely removed without self-contamination when removing the PPE. These gloves should at the very least have extended cuffs.

Which food items, if chosen by a new unlicensed assistive personnel (UAP), would indicate to the nurse that the UAP understands a clear liquid diet? 1. White grape juice 2. Gelatin 3. Vanilla pudding 4. Lemon Popsicle 5. Fat free Broth 6. Tea with honey

1., 2., 4., 5., & 6. Correct: A clear liquid diet is made up of only clear fluids and foods that are clear fluids when they are at room temperature. These choices are considered to be clear liquids. 3. Incorrect. This is considered appropriate for a full liquid diet.

A client who is at high risk for developing a stroke has been advised to follow a Mediterranean type diet by the primary healthcare provider. Which food choices, if selected by the client, would indicate to the nurse that the client understands this diet. Select all that apply 1. Grilled eggplant 2. Purple grape juice 3. Bacon 4. Cashews 5. Skim milk 6. Salmon

1., 2., 4., 5., & 6. Correct: It is reasonable to counsel clients to follow a Mediterranean-type diet over a low-fat diet. The Mediterranean type diet emphasizes vegetables, fruits, and whole grains and includes low fat dairy products, poultry, fish, legumes, and nuts. It limits intake of sweets and red meats. 3. Incorrect: Substitute fish and poultry for red meat. When eaten, make sure it's lean and keep portions small (about the size of a deck of cards). Also avoid sausage, bacon and other high-fat meats.

A nurse is planning a teaching session for a group of clients diagnosed with irritable bowel syndrome. What points should the nurse include to help the clients control symptom flare-ups? Select all that apply 1. If you are constipated, try to make sure you have breakfast. 2. Avoid low fat foods. 3. If you think a certain food is a problem, try cutting it out of your diet for about 12 weeks. 4. Drinks containing caffeine are likely to contribute to symptoms. 5. Foods such as broccoli and cabbage are good sources of fiber.

1., 3. & 4. Correct: If you are constipated, try to make sure you eat breakfast, as this is the meal that is most likely to stimulate the colon and give you a bowel movement. If you think a certain food is a problem, try cutting it out of your diet for about 12 weeks. (If you suspect more than one, cut out one at a time so you know which one causes you problems.) If there's no change, go back to eating it. The foods most likely to cause problems are: Insoluble (cereal) fiber; Coffee/caffeine; Chocolate; Nuts 2. Incorrect: Avoid meals that over-stimulate the gut, like large meals or high fat foods. 5. Incorrect: Broccoli and cabbage are common gas-producing foods that can cause abdominal distention and flatulence.

A client requires external radiation therapy. The nurse knows external radiation may cause which problems? Select all that apply 1. Pancytopenia 2. Leukocytosis 3. Erythema 4. Fever 5. Fatigue

1., 3. & 5. Correct: Effects of radiation therapy include, but are not limited to pancytopenia (marked decrease in the number of RBCs, WBCs and platelets), erythema (redness of the skin), and fatigue.2. Incorrect: Leukocytosis is an increase in WBCs. External radiation causes pancytopenia which is a decrease in the number of blood cells including WBCs. 4. Incorrect: Fever is not typically seen with external radiation.

Which nursing intervention should the nurse include when planning care for a client with Parkinson's disease? Select all that apply 1. Encourage high fiber diet with increased fluid intake. 2. Schedule activities early in the morning. 3. Help client perform stretching and strengthening exercises. 4. Teach client to sit upright at 90 degree angle when eating. 5. Maintain a high protein diet while taking levodopa.

1., 3., & 4. Correct: These are appropriate interventions for a client with Parkinson's disease. Constipation is a common problem in clients with Parkinson's disease. Increasing dietary fiber and fluid intake is useful in managing this problem. These clients have impaired mobility, so stretching and strengthening exercises are necessary to keep muscles working as long as possible. Loss of control of the mouth and throat muscles causes difficulty swallowing. Sitting upright at 90 degrees makes chewing and swallowing easier by using your posture to assist with the process. 2. Incorrect: Appointments and activities are best scheduled for late morning so that the client will not be rushed in performing self-care. 5. Incorrect: For clients on levodopa, explain that they shouldn't take this drug with high-protein meals as this can reduce its absorption and availability. Caution them to avoid foods and supplements high in pyridoxine (vitamin B6), which can inhibit the drug's action.

A client with type 2 diabetes, who is noncompliant, has a HbA1c of 8%. The finger stick blood sugar is 218 mg/dL (12.1 mmol/L) at 0900. The current medications prescribed are metformin and exenatide. Based on this data, what teaching should the nurse reinforce? Select all that apply 1. Nutritional counseling to help improve diet compliance 2. HbA1c measures glycemia control over a period of 1 month 3. Blood glucose testing 4. Vigorous exercise plan to improve glucose control 5. Without glycemic control, eye complications can occur

1., 3., & 5. Correct: The goal of therapy is to have a HbA1c <7.0% Nutritional teaching to promote diet compliance should be tried first because this clients HbA1c is 8% and blood sugar is 218 mg/dL (12.1 mmol/L). Reinforce need to monitor glucose levels several times a day, before meals and at bedtime. Have client keep results in a log. Eye complications include glaucoma, cataracts, retinopathy, blindness. 2. Incorrect: HbA1c measures glycemic control over a period of 3 months. 4. Incorrect: Physical activity under the supervision of the primary healthcare provider is appropriate teaching. Vigorous exercise is not key to improving blood sugar. Exercise does not have to be intense. A normal exercise plan is recommended based on the client's ability.

Which health promotion instructions should the nurse provide to a client diagnosed with cirrhosis? 1. Use a shower chair when performing hygiene. 2. Limit alcohol intake. 3. Stop any activity that causes dizziness. 4. Calculate daily sodium intake. 5. Proper hand hygiene.

1., 3., 4., & 5. Correct: Using a shower chair while showering and performing hygiene will help to save energy. Stop any activity that causes chest pain, a marked increase in shortness of breath, dizziness, or extreme fatigue or weakness. High sodium promotes fluid volume excess. The client should maintain a low sodium intake. Proper hand hygiene prevents infection. 2. Incorrect: The client must stop drinking alcohol to halt the progression of cirrhosis.

What nursing interventions should the nurse include when planning care for a client admitted with Guillain-Barre' Syndrome? Select all that apply 1. Monitor for contractures. 2. Place prone for 30 minutes, 4 times per day. 3. Provide therapeutic massage for pain relief. 4. Teach range of motion exercises. 5. Provide high protein meals 3 times a day. 6. Refer to physical therapist.

1., 3., 4., & 6. Correct: This client will have progressive weakness and paralysis. Contractures and pressure ulcers need to be prevented through ROM exercises and frequent turning. Muscle spasms and pain can be relieved by therapeutic massage, imagery, diversion, and pain medication. 2. Incorrect: The client will need to be repositioned every 2 hours to prevent pressure sores and pneumonia and atelectasis. Elevate the head of the bed to help with lung expansion. Prone will interfere with lung expansion ability. 5. Incorrect: Encourage small, but frequent meals that are both well-balanced and nourishing.

A client who needs to have a stool specimen for an occult blood test is instructed by the nurse to avoid which substances two hours prior to testing? Select all that apply 1. Liver 2. Tomato 3. Ibuprofen 4. Sardines 5. Ascorbic acid

1., 3., 4., 5. Correct: The following foods can cause a false-positive reading: red meats, liver, turnips, broccoli, cauliflower, melons, salmon, sardines, and horseradish. Medications altering the test include aspirin, ibuprofen, ascorbic acid, indomethacin, colchicines, corticosteroids, cancer chemotherapeutic agents, and anticoagulants. Ingestion of vitamin rich foods can cause a false negative result. 2. Incorrect: A tomato is not on the food list for false-positive reading and do not have to be avoided.

A nurse is planning to discuss steps that senior citizens can take to keep the brain healthy. What should the nurse include? Select all that apply 1. Memorize poetry. 2. Eat foods low in Omega 3, fatty acids. 3. Brush teeth with nondominant hand. 4. Do crossword puzzles. 5. Learn a new language. 6. Volunteer.

1., 3., 4., 5., & 6. Correct: All of these activities exercise the brain and increase mental functioning. 2. Incorrect: Brain-boosting food is any food high in Omega 3 fatty acids, which has been linked to a lower risk of dementia and improved focus and memory.

A client returns to the unit after a liver biopsy. Which nursing interventions would the nurse implement? 1. Put a pillow under the costal margin. 2. Place in the right side lying position. 3. Perform passive range of motion exercises to right shoulder. 4. Take vital signs every 10 - 15 minutes for first hour. 5. Instruct the client to avoid strenuous exercise for 1 month.

1.,2., & 4 Correct: The client is placed on the right side and a pillow placed under the costal margin. The pillow will place additional pressure on the rib cage which will assist with applying pressure to the liver capsule. By positioning the client on the right side, the liver capsule at the site of the biopsy is compressed against the chest wall. If the puncture site is not compressed, there is the possibility that blood or bile will leak from the puncture site. The vital signs are measured at 10 - 15 minute intervals for the first hour. Variations of the vital signs will indicate complications such as bleeding, severe hemorrhage, and bile leakage. 3. Incorrect: Passive range of motion exercises is not correct. The shoulder is not placed in a position during and after the biopsy to warrant passive exercises to the shoulder. 5. Incorrect: The client should be instructed to avoid strenuous exercise for 1 week not 1 month. The strenuous exercise is restricted to 1 week to prevent the possibility of liver bleeding.

A client has been admitted to the medical unit after sustaining a stroke. The admitting nurse initiates a nursing diagnosis of unilateral neglect related to a decrease in visual field and hemianopia from cerebrovascular problems as evidenced by consistent inattention to stimuli on the affected side. What nursing interventions should the nurse initiate for this client? Select all that apply 1. Instruct client to scan from left to right to visualize the entire environment. 2. Encourage client to practice exercises independently. 3. Position bed in room so that individuals approach the client on the unaffected side. 4. Apply splints to achieve stability of affected joints. 5. Touch unaffected shoulder when initiating conversation with client. 6. Position personal items within view on the unaffected side.

1., 3., 5., & 6. Correct: Instructing the client to scan from left to right will help the client to visualize the entire environment. The client has to be reminded to do this since only one side of the client's visual field is working. By positioning the bed so that individuals approach the client from the unaffected side and by touching the client on the unaffected shoulder, the client is not surprised or frightened when realizing someone is in the room. Placing personal items where the client can see them will allow the client to use the material. Then gradually move personal items and activity to the affected side as the client demonstrates an ability to compensate for neglect. 2. Incorrect: Practicing exercises independently focuses on impaired physical mobility rather than unilateral neglect. 4. Incorrect: Applying splints to affected joints focuses on impaired physical mobility rather than unilateral neglect.

The nurse is providing teaching for a client who is being scheduled for outpatient 24 hour electrocardiogram monitoring using a Holter monitor. What should the nurse tell the client to avoid while monitoring is in progress? Select all that apply 1. Taking a shower or bath 2. Performing daily exercises 3. Working around high voltage equipment 4. Being screened at airport security 5. Eating foods that are sources of potassium

1., 3., and 4. Correct: The nurse should teach this client to continue the usual activities while wearing the monitor with a few exceptions. The monitor should be kept dry to ensure that it functions properly. The client should avoid taking a shower or bath or swimming while wearing the monitor. The electrodes could also become detached from the skin if they get wet, which would also interfere with the accuracy of the reading. The client should be advised to not work around high voltage equipment because areas of high voltage can interfere with the function of the electrocardiogram monitoring. In addition, magnetic fields, such as those used for airport screenings, can interfere with the function of the Holter monitor and should be avoided. 2. Incorrect: This client should be encouraged to continue regular routine unless otherwise directed by the primary healthcare provider. The client can perform the usual daily exercise, but should be advised to avoid activities that may cause excessive perspiration that could lead to the electrodes becoming loosened from the skin. 3. Incorrect: There are generally no dietary restrictions while wearing the Holter monitor unless otherwise prescribed by the primary healthcare provider.

A client with renal failure has returned to the unit post kidney transplant. Which postoperative interventions should the nurse provide? Select all that apply 1. Administer furosemide. 2. Maintain fluid replacement at 150 ml per hour for 8 hours. 3. Measure abdominal girth every 24 hours. 4. Weigh daily. 5. Measure urine output every 30 - 60 minutes.

1., 4., & 5. Correct: Diuretics are administered to promote postoperative diuresis. Daily weights are done to make sure there is not rapid weight gain which is a pertinent of fluid retention. Careful and frequent assessment of UOP helps determine fluid balance and transplant function. Oliguria is an early sign of acute tubular necrosis and should be detected as soon as possible post-op.2. Incorrect: Fluid replacement is generally calculated to replace urine output over the previous 30-60 minutes, milliliter for milliliter. The stem of the question does not give you enough information about the client to know that this rate of fluid replacement is safe.3. Incorrect: Signs of hemorrhage after this surgery include swelling at the op site, increased abdominal girth, signs of shock and decreased level of consciousness. Assessment for signs of hemorrhage should be done much more frequently than every 24 hours.

What interventions should the nurse include in the care plan of a client admitted with Guillain-Barre syndrome? Select all that apply 1. Assess for descending paralysis. 2. Keep a sterile tracheostomy at the bedside. 3. Monitor for heart rate above 120/min. 4. Maintain in side-lying, supine position. 5. Have client perform active range of motion (ROM) every 2 hours while awake.

2. & 3. Correct: This client is at risk for respiratory paralysis as the disease progresses. An emergency tracheostomy may need to be performed so the nurse should watch out for imminent signs of respiratory failure. Signs include heart rate that is more than 120 bpm or lower than 70 bpm and respiratory rate of more than 30 bpm. The nurse should assess for signs of respiratory distress and prepare for intubation if needed. 1. Incorrect: Ascending paralysis should be assessed for with this disease. Paralysis begins in the lower extremities and moves upward. 4. Incorrect: The client should be assisted to a position with head of bed elevated for full chest excursion. 5. Incorrect: The nurse should perform passive range of motion exercises. Active exercise should be avoided during the acute phase as the client is easily fatigued and muscles are weak. Passive ROM stimulates circulation, improves muscle tone and increases joint mobilization.

The nurse assesses a client post thyroidectomy for complications by performing which assessment? 1. Accucheck 2. Chovostek's 3. Ballottement 4. Ice water colonic

2. Correct: A positive Chovostek's and Trousseau's is indicative of tetany and low calcium. This can occur when a couple of parathyroids are accidently removed when the thyroid is removed. 1. Incorrect: Accucheck assesses for blood glucose levels, which is not the problem post thyroidectomy. 3. Incorrect: This assessment technique is used in examining the abdomen when ascites is present. It is done by palpating the abdomen to detect excessive amounts of fluid (ascites). 4. Incorrect: If you have never heard of it, no one else has either. The phrase implies using ice water to cleanse the colon and this would never be a good thing, especially for someone post thyroidectomy that would be intolerant to extremes in temperature.

A female client receiving chemotherapy for breast cancer reports vomiting, stomatitis, and a 10 pound weight loss over the past month. The primary healthcare provider orders an antiemetic and daily mouthwashes. When the home care nurse evaluates the client one week later, what change described by the client would best indicate improvement? 1. Eating three meals daily. 2. Weight gain of two pounds. 3. No further mouth pain. 4. Improved skin turgor.

2. Correct: Chemotherapy typically causes gastrointestinal disturbances severe enough to interfere with a client's ability to eat or absorb nutrients. A ten pound weight loss over one month is significant but expected because of the reported vomiting and stomatitis. A weight gain of two pounds in a week would be the best specific indicator of improvement. 1. Incorrect: The ability to eat three meals daily does not mean that the client is actually absorbing those nutrients successfully. This option suggests that the antiemetic is working well, but there is not enough evidence to demonstrate significant client improvement. 3. Incorrect: The client's denial of any further mouth pain signifies that the mouthwashes have decreased mouth inflammation and stomatitis. While this is a positive change in the client's condition, it is not the best evidence noted by the nurse. 4. Incorrect: Skin turgor specifies the hydration status of a client. Since this client had previously been vomiting, improved skin turgor would indicate the antiemetic is working well and the client is able to retain fluids. While this is a positive change, it is not the most significant indicator of client improvement.

Which signs and symptoms will the nurse include when teaching a client about indicators of recurrent nephrotic syndrome? Select all that apply 1. Dysuria 2. Hematuria 3. Foamy urine 4. Periorbital edema 5. Weight loss

3. & 4. Correct: Foamy urine, which may be caused by excess protein in the urine, is seen with nephrotic syndrome. Swelling (edema), particularly around the eyes (periorbital) and in the ankles and feet, is a symptom.1. Incorrect: Dysuria would be a symptom of disorders such as kidney stone or UTI, rather than nephrotic syndrome.2. Incorrect: Proteinuria rather than hematuria is seen.5. Incorrect: Weight gain is seen with renal disorders due to poor renal function and increased fluid volume.

A client with a diagnosis of endocarditis and a new peripherally inserted cential catheter (PICC) line has been discharged home to receive daily intravenous antibiotics for six more weeks. The home health nurse is making an assessment visit today. What instruction by the nurse is most important initially? 1. Take antibiotics before dental procedures. 2. Brush and floss teeth at least twice daily. 3. Report any flu like symptoms immediately. 4. Include rest periods throughout the day.

2. Correct: Poor dental hygiene is one of the chief causes of endocarditis in adults, leading to growth of vegetation on heart valves, emboli, strokes, or even death. Instructions on proper oral care is considered primary or preventative teaching and encourages the client to take an active role in personal health care. Decreasing mouth bacteria or disease will decrease the potential for a reoccurrence of endocarditis. 1. Incorrect: Although primary healthcare providers may order antibiotics prior to a dental visit, it depends on what procedure the dentist is going to perform. Invasive mouth procedures where bleeding is likely generally require pre-visit antibiotics. However, this is not the most important information by the nurse initially. 3. Incorrect: Flu like symptoms are an indication of a possible exacerbation or reoccurrence of endocarditis. The client would be instructed to report such signs as fever, chills, malaise, or night sweats immediately to the primary healthcare provider. While it is important for the client to understand what to report, preventative measures are more important at this time. 4. Incorrect: Infection within the heart is very serious and, despite aggressive treatment, may have lasting effects on the client's cardiovascular system. Decreasing the workload of the heart during treatment and recovery time would certainly assist with the healing process. However, the need for frequent rest periods throughout the day is determined by a variety of factors, such as the client's age and morbidity factors, general health, amount of damage to the heart, and response to antibiotics. Rest is not the most important instruction the nurse must present initially.

A RN is observing an unlicensed nursing personnel (UAP) feed a client who is on aspiration precautions. Which action by the UAP would require the nurse to intervene? 1. Elevating the head of the bed to a 90 degree angle 2. Instructing the client to lean the head back slightly when swallowing. 3. Adding a thickening agent to liquids. 4. Feeding the client small amounts of food per bite.

2. Correct: This is an incorrect action, and needs intervention by the nurse. The chin should be flexed to prevent the risk of aspiration. A chin down or chin tuck maneuver is widely used in dysphagia treatment to prevent aspiration. 1. Incorrect: This is a correct action. The head of the bed should be elevated which aids in esophageal peristalsis and swallowing is aided by gravity. 3. Incorrect: This is a correct action. Thickened liquids are easier to swallow without aspirating. Drinking liquids thickened will help to prevent choking and stops fluid from entering the lungs. 4. Incorrect: This is a correct action by the UAP, so the nurse does not need to intervene. Smaller amounts of food can be chewed more thoroughly and swallowed with less risk for aspiration.

A client with a deep partial-thickness burn to the right forearm has returned from surgery with a skin graft to the burned area. Which graft site intervention would the nurse implement within the first 24 hours? 1. Monitor temperature every 12 hours. 2. Position arm to prevent pressure to the graft site. 3. Prepare to change the 1st dressing within 24 hours. 4. Perform passive range of motion exercises to the right arm.

2. Correct: The arm should be situated so there is no compression on the graft site. Applying pressure to the graft may cause the graft to move which may result in damage to the graft site. 1. Incorrect: The temperature should be monitored at least every 8 hours or less. An elevated temperature may indicate an infection under the graft or at another area. If the temperature is elevated, the source of the infection should be addressed. 3. Incorrect: The first dressing change is planned 2 to 5 days after surgery. The dressing is not changed in the first 24 hours to prevent any disturbance to the skin graft. 4. Incorrect: The burn is located on the forearm. Passive range of motion exercises are implemented to improve joint function. The client is not having difficulties with the wrist, elbow, or shoulder joints. The movement to the arm should be done cautiously to prevent any damage to the graft.

A client has been trying to implement a low fat diet for prevention of heart disease and enhancement of weight loss. He further reports that his wife shows her love by preparing rich foods and pastries. Which action should the nurse make? 1. Suggest that the client prepare all meals at home. 2. Schedule a meeting with husband and wife to discuss diet and health. 3. Suggest that the client limit intake to one serving of each food at meals. 4. Ask the client to give his wife a cookbook with low fat recipes.

2. Correct: The meeting with the wife and husband together may help to gain the support of the wife. She may not realize that meal preparation is actually serving as a barrier to successful change. Also, the importance of the opinions and behaviors of the wife are important to the client as he tries to engage in long-term behavioral change. 1. Incorrect: This intervention may actually increase barriers to change because the wife's feeling and support are necessary to maintain long-term change.3. Incorrect: While this practice may reduce the intake of fat, the issue of spousal support should be addressed.4. Incorrect: Open discussion with the wife about the need for low-fat meals is essential.

A client with diabetes has a history of ignoring the primary healthcare provider's prescription for daily medication management of the illness. The client has been working toward a health promotion goal of increased adherence to prescribed medication regimen. Which outcome suggests that the client has met the health promotion goal? 1. Client has lost five pounds. 2. Client takes medication as prescribed. 3. Client has been hospitalized twice for complications of diabetes. 4. Client walks one mile per day.

2. Correct: The outcome directly addresses medication adherence, the major focus of the health promotion plan. 1. Incorrect: This is a positive outcome; however, the focus is on medication adherence.3. Incorrect: This outcome would indicate possible non-adherence to the medication regimen.4. Incorrect: While this is a positive outcome for anyone's health, the focus is medication adherence.

What preferred lab value would the nurse expect to see the primary healthcare provider prescribe for a client admitted with generalized malnutrition? 1. Albumin 2. Prealbumin 3. Iron 4. Calcium

2. Correct: The preferred lab value to screen for generalized malnutrition is prealbumin. This assessment is preferred because it decreases more quickly when nutrition is not adequate. 1. Incorrect: Albumin is a major serum protein that is below normal in clients who have inadequate nutrition. However, it can take weeks to drop. 3. Incorrect: Low serum iron and anemia indicate an iron deficiency. Again, the prealbumin will decrease sooner than other lab values that assess nutrition level. 4. Incorrect: Older women may have low calcium levels which place them at risk for bone demineralization. But, prealbumin provides more data on generalized nutrition.

The nurse is preparing to educate a group of clients on how to decrease the risk of developing recurrent renal calculi. What topics should the nurse include? Select all that apply 1. High-purine foods to consume 2. Discuss diuretic use to prevent urinary stasis 3. Straining urine with each void 4. Maintaining a daily water intake of at least 2 liters 5. Foods low in calcium

2., 4., & 5. Correct: Diuretics are often used to prevent urinary stasis and further calculus formation. Thiazides decrease calcium excretion into the urine. Daily fluid intake should be 2-3 liters per day to ensure good renal function. Most stones are calcium stones, so decrease calcium in the diet to reduce the chance of calcium stones.

A client's absolute neutrophil count (ANC) is 750/mm3. Which measures should the nurse take to protect the client? 2., 3. & 4. Correct: If a client's ANC is less than 1000/mm3, the client is at risk for infection. Instructing the client to wear a mask outside of the hospital room protects the client from infection. The soil in fresh flowers and plants can carry bacteria and fungi, which can cause infection. Performing hand hygiene is the best way to prevent the spread of infection. 1. Incorrect: Not allowing the client to shave would be an appropriate intervention for someone with a low platelet count. 5. Incorrect: Not allowing the client to floss the teeth would be an appropriate intervention for someone with a low platelet count. The client needs good oral care to prevent infections in the mouth.

2., 3. & 4. Correct: If a client's ANC is less than 1000/mm3, the client is at risk for infection. Instructing the client to wear a mask outside of the hospital room protects the client from infection. The soil in fresh flowers and plants can carry bacteria and fungi, which can cause infection. Performing hand hygiene is the best way to prevent the spread of infection. 1. Incorrect: Not allowing the client to shave would be an appropriate intervention for someone with a low platelet count. 5. Incorrect: Not allowing the client to floss the teeth would be an appropriate intervention for someone with a low platelet count. The client needs good oral care to prevent infections in the mouth.

What signs/symptoms would the nurse expect to assess in a client diagnosed with acute pericarditis? Select all that apply 1. Petechiae on trunk 2. Muffled heart sounds 3. Pericardial friction rub 4. Pulsus paradoxus 5. Chest pain on deep inspiration

2., 3., & 5. Correct: Muffled heart sounds are indicative of pericarditis. Fluid is between the heart and the chest wall; heart sounds are lowered and distant. A pericardial friction rub is a classic symptom of acute pericarditis. Chest pain is the most common symptoms of pericarditis, and is aggravated by deep inspiration, coughing, position change, and swallowing. 1. Incorrect: Petechiae on the trunk, conjunctiva, and mucous membranes are indicative of endocarditis. 4. Incorrect: Pulsus paradoxus is an exaggerated decrease of systolic blood pressure during inspiration exceeding 12 mm Hg. It is the hallmark of cardiac tamponade.

The nurse is cleaning and dressing a foot ulcer of a diabetic client. Which actions are appropriate? Select all that apply 1. Uses a clean basin and washcloth to clean the ulcer. 2. Wears sterile gloves to clean the ulcer. 3. Cleans ulcer with normal saline. 4. Warms saline bottle in microwave for 1 minute. 5. Cleans ulcer in a full circle, beginning in the center and working toward the outside.

2., 3., & 5. Correct: The nurse needs to wear sterile gloves when cleaning the wound. Normal saline solution is the preferred cleansing agent because, as an isotonic solution, it doesn't interfere with the normal healing process. Gently clean the wound in a full or half circle, beginning in the center and working toward the outside. 1. Incorrect: Sterile supplies should be used with this procedure because the client is at risk for infection and gangrene. Gauze and salve should be used instead of a wash cloth. 4. Incorrect: Before you start, make sure the cleansing solution is at room temperature. Do not heat in the microwave. It could scald the client!

An elderly client living in a long-term care facility fell 8 hours ago causing a laceration on the occipital area of the skull and steri-strips were applied for closure. Which signs/symptoms would indicate to the nurse that the client should be transferred to the emergency department? Select all that apply 1. Purposeful movement. 2. Sudden emotional outbursts. 3. Client report of blurred vision. 4. Pupils equal, react to light, and accommodation. 5. Bright red blood oozing from the wound. 6. Headache unrelieved by acetaminophen.

2., 3., & 6. Correct: Signs/symptoms of increased ICP include: excessive sleepiness, inattention, difficulty concentrating, impaired memory, faulty judgment, depression, irritability, emotional outbursts, disturbed sleep, diminished libido, difficulty switching between two tasks, and slowed thinking. Abnormalities in vision and extraocular movements occur in the early stages of increased ICP. A headache that is unrelieved by acetaminophen would warrant further investigation. 1. Incorrect: This is a normal response and does not warrant further investigation. 4. Incorrect: This is a normal response and does not warrant further investigation. 5. Incorrect: The scalp is very vascular and oozing would be expected. Apply pressure to stop bleeding.

The nurse is teaching a group of adults how to check skin lesions for signs of melanoma. What should the nurse include? 1. Symmetrical shape 2. Multiple colors with a lesion 3. Odd looking lesion 4. Poorly defined border of lesion 5. Diameter of lesion 6 mm

2., 3., 4., & 5. Correct: Multiple colors, varying colors from one area to another or uneven distribution of color could mean cancer. Some moles don't exactly fit the criteria for ABCDs of melanoma, but they can be cancerous nonetheless. If you see a mole that looks odd, or you see sudden changes in size, color or shape, get it checked. An irregular scalloped or poorly defined border of the growth could be a sign of melanoma. The diameter of a melanoma is usually bigger than 1/4 inch or 6 mm when diagnosed, but they can be smaller. 1. Incorrect: An asymmetrical shape can be an indicator of melanoma.

Which signs/symptoms noted by the nurse would support a client history of chronic emphysema? Select all that apply 1. Atelectasis. 2. Increased anteroposterior (AP) diameter. 3. Breathlessness. 4. Use of accessory muscles with respiration. 5. Leans backwards to breathe. 6. Clubbing of fingernails

2., 3., 4., & 6. Correct: Emphysema is described as a permanent hyperinflation of lung beyond the bronchioles with destruction of alveolar walls. Airway resistance is increased, especially on expiration. Inspection reveals dyspnea on exertion, barrel chest (anteroposterior diameter is equal to transverse diameter), tachypnea, and use of accessory muscles with respiration. Clubbing of fingernails is due to chronically decreased oxygen levels. 1. Incorrect: Atelectasis is collapse of alveolar lung tissue, and findings reflect presence of a small, airless lung. This condition is caused by complete obstruction of a draining bronchus by a tumor, thick secretions, or an aspirated foreign body, or by compression of lung.

A client arrives to the emergency department after an altercation resulting in a knife wound to the abdomen. The nurse makes the following observation. Which intervention should the nurse perform? 1. Place the client in trendelenburg position. 2. Instruct the client to lie quietly in a low Fowler's position. 3. Apply abdominal binder to gently place the intestines back in the abdominal cavity. 4. Cover exposed intestine with sterile dressings moistened with sterile saline. 5. Notify the surgeon at once.

2., 4. & 5. Correct: Low Fowler's position and staying calm and quiet help to minimize protrusion of body tissues. Cover exposed intestines with sterile dressings moistened with sterile saline solution. Have someone notify the surgeon at once and you stay with the client and stay calm. 1. Incorrect: The client should be placed in the low Fowler's position and instructed to lie quietly. These actions minimize protrusion of body tissues. 3. Incorrect: Never push eviscerated abdominal contents back! And do not apply pressure with an abdominal binder. This client needs to go back to surgery.

A client admitted to a long-term care facility is legally blind and partially deaf. How would the nurse best provide for the client's safety in the event of an emergency? Select all that apply 1. Have roommate lead client out of the room to safety area. 2. Assign a specific UAP every shift to escort client to safety. 3. Research established protocols utilized by emergency groups. 4. Discuss best communication methods with client and family. 5. Plan for the supervisor to be responsible for evacuating the client.

3 & 4. Correct: When faced with a new or challenging situation involving client safety, the nurse manager should employ the Nursing Process to assess needs and collect contributing data. Asking for input from emergency preparedness groups, such as the Red Cross or FEMA, could provide ideas about assisting individuals with sensory deficits. Secondly, the nurse should discuss the situation with both client and family to determine appropriate methods of communicating with client, particularly in an emergency situation. 1. Incorrect: The responsibility for client safety should never be placed on a roommate or even family members. Staff should be accountable for client safety at all times. 2. Incorrect: Assigning a specific UAP each shift to locate and escort client to a safe area would be confusing. Changing protocols every shift creates a hazardous situation for staff and clients. 5. Incorrect: The facility supervisor is responsible for all aspects of an emergency, including activation of alarms, coordinating evacuation of staff and clients, and initiating facility emergency protocols such as closing fire doors or turning off oxygen valves. It would not be safe for the supervisor to also be responsible for a single individual.

A client arrives at the emergency room with active gastrointestinal bleeding. What is the most important nursing action? 1. Treat the cause of the bleeding. 2. Record the amount of blood loss. 3. Initiate an intravenous access line. 4. Prepare client for stat endoscopy.

3. CORRECT: The client has active gastrointestinal bleeding, which can quickly lead to hypovolemic shock. Active bleeding would be treated with fluids, and in certain cases, blood products. Establishing an IV site allows for immediate initiation of treatment before veins vasoconstrict and become too difficult to access secondary to shock. 1. INCORRECT: While it is crucial to find and treat the cause of the bleeding, diagnosing is the responsibility of the primary healthcare provider. Because this client is experiencing internal bleeding, further tests may be needed to determine the source of the hemorrhage. 2. INCORRECT: Gastrointestinal bleeding is very difficult to measure since there is no effective way to collect the fluid. The primary healthcare provider could order a hemoglobin and hematocrit but that does not precisely measure the actual amount of blood loss. 4. INCORRECT: An endoscopy is an internal examination of a portion of the gastric system. However, proper preparation requires the client to be NPO for hours in order to properly visualize that system. This is not the most important nursing action.

A client had an open cholecystectomy several days ago. What finding by the nurse should be reported to the primary healthcare provider immediately? 1. Respiratory rate of 30 2. Blood pressure reading of 104/50 3. Incisional pain with foul, green drainage 4. Urinary output of 75 mL straw colored urine

3. CORRECT: The client is having incisional pain, which by itself could be expected following an open cholecystectomy. However, there should never be any foul, green drainage from an incision, as this indicates a post-operative infection. The nurse should report this immediately to the primary healthcare provider. 1. INCORRECT: Although this respiratory rate seems slightly on the elevated side, this client has had recent surgery and is now having some complications. Combined with the pain, this rapid respiratory rate would be expected. 2. INCORRECT: There is no baseline data provided regarding this blood pressure data. Without a reference to a client's previous blood pressure, it is impossible to form any opinion about this reading. We worry about a systolic BP of 90. 4. INCORRECT: The information provided in the question does not give any parameters by which to evaluate the urine. Straw colored urine is a normal finding; however, there is no indication regarding the length of time it took to accumulate 75 mL of urine. Therefore, no decision can be formed about this finding

Following surgery, a client has an indwelling urinary catheter attached to a collection bag. The nurse empties the collection bag at 0900. At the change of shift at 1500, the collection bag contains 100 mL of urine. The system has no obstructions to urinary flow. What would be the nurse's most appropriate initial response? 1. Elevate the head of the client's bed. 2. Start giving the client 8 ounces of oral fluid per hour. 3. Check circulation and take the vital signs of the client. 4. Continue monitoring, because this is an expected finding.

3. Correct: A urine output (U/O) of 100 mL over a 6 hour period is dangerously low. This client could be experiencing hypovolemic shock. In clients who are "shocky", the kidneys stop making urine to try to hold on to what little volume the body has left. The nurse is checking the vital signs for low BP and increased HR, indicators of hypovolemic shock. Also, when the urine output is this low, the client is at risk for renal failure. 1. Incorrect: Elevating the head of the client's bed is a good choice when the client is having difficulty breathing, but not here. Raising the HOB will cause the BP to drop lower. Clients in shock should be supine. 2. Incorrect: Normally, pushing fluids is a good choice if the urine output were low. 100 mL over six hours requires more aggressive treatment to combat shock. 4. Incorrect: This is not an expected finding. Urine output less than 240 mL in an eight hour time frame should alert the nurse to a serious problem such as shock.

What is the most effective method of stroke prevention that the nurse should teach to the public? 1. Administering platelet inhibitors to prevent clot formation. 2. Undergoing transluminal angioplasty to open a stenosed artery and improve blood flow. 3. Maintaining normal weight, exercising, and controlling comorbid conditions. 4. Administering tissue plasminogen activator (tPA).

3. Correct: Although administering platelet inhibitors, tPA, and undergoing transluminal angioplasty may improve cerebral blood flow, the goals of stroke prevention include health promotion for the healthy individual and education and management of modifiable risk factors to prevent a stroke. Health promotion focuses on a healthy diet, weight control, regular excercising, no smoking, limited alcohol consumption and routine health assessments. 1. Incorrect: This is not the most effective method to prevent a stroke. Anti platelet drugs are usually the chosen treatment to prevent stroke in clients who have had a TIA. 2. Incorrect: This is not the most effective method to prevent a stroke. Transluminal angioplasty is an invasive procedure to improve blood flow. 4. Incorrect: This is not the most effective method to prevent a stroke. tPA is administered IV to reestablish blood flow through a blocked artery in a client with acute onset of ischemic stroke symptoms.

The nurse sees the following rhythm on the cardiac monitor for a client recovering from a myocardial infarction. What would be the nurse's first action upon entering the client's room? Exhibit: VF 1. Attempt defibrillation 2. Begin CPR 3. Assess for carotid pulse 4. Administer lidocaine

3. Correct: Although the rhythm strip looks like ventricular fibrillation, you must first check the client. Assess for consciousness, airway, breathing, circulation first. 1. Incorrect: Assess the client first. Do not rely on the strip alone. It may be artifact. If there is no pulse, then you defibrillate. 2. Incorrect: Assess the client first. Defibrillate, then CPR. 4. Incorrect: While CPR is in progress after defibrillation, start IV, if one is not available, then give lidocaine.

The nurse is caring for a client admitted with heart failure associated with an acute MI. At which time point did the nurse begin to intervene incorrectly? Exhibit Nitroglycerin 25 mg in 250 mL D5W IV administered per protocol. 1. 1115 2. 1120 3. 1125 4. 1130IV D5W at 20 mL/hr

3. Correct: At 1125, the nurse failed to follow protocol for nitroglycerin infusion. The nurse increased the IV rate by 6 mL/hr (going from 10-20 mcg/min). 1. Incorrect: The nurse mixed the nitroglycerin appropriately and connected the tubing at the correct IV site. The infusion rate was started at 3 mL/hr which delivered the appropriate starting dose at 5 mcg/min. 2. Incorrect: At 1120 the client is still hurting and the BP is above 120 systolic, so the nitroglycerin infusion can be increased by 5 mcg/min which would increase the rate to 6 mL/hr. 4. Incorrect: At 1130 the client is still hurting and the BP is above 120 systolic, so the nitroglycerin infusion can be increased by 5 mcg/min which would increase the rate to 15 mL/hr.

What should the nurse teach a client about testicular self examination? 1. This exam should be performed bi-annually. 2. The exam should be performed during a cold shower. 3. Gently roll each testicle with slight pressure between the fingers. 4. The epididymis should feel like a hard, knotty rope.

3. Correct: Examine one testicle at a time. Use both hands to gently roll each testicle, with slight pressure, between the fingers to feel for lumps, swelling, soreness or a harder consistency. 1. Incorrect: All men 15 years and older need to perform this examination monthly. 2. Incorrect: The exam should be performed during or right after a warm shower or bath when the the scrotum is less thick. 4. Incorrect: The epididymis should feel soft, rope like, and slightly tender to pressure. It is located at the top of the back part of each testicle. It is not a lump.

A middle-aged client has a strong positive family history of type 2 diabetes mellitus. What should the nurse teach the client regarding the best method to prevent or delay the development of this disease? 1. Test serum glucose values monthly. 2. Avoid starches and sugars in the diet. 3. Obtain a normal body weight and exercise regularly. 4. Maintain a normal serum lipid panel.

3. Correct: Genetics and body weight are the most important factors in the development of type 2 diabetes mellitus. The client cannot alter his genetics. Therefore, a normal body weight is imperative. Regular exercise reduces insulin resistance and permits increased glucose uptake by cells. This serves to lower insulin levels and reduce hepatic production of glucose. 1. Incorrect: Monthly glucose monitoring is not sufficient. It will tell you when the client becomes a diabetic but will not prevent it from happening. 2. Incorrect: Starch and sugar intake should be decreased, not avoided. 4. Maintaining a normal serum lipid panel may not be achievable in some clients, but it is always the goal. Medication may be needed.

What information should be included when a nurse is teaching a group of college students about the transmission of hepatitis B and human immunodeficiency virus (HIV)? 1. HIV is transmitted via toilet seats whereas hepatitis B is not. 2. HIV is transmitted by sexual contact whereas hepatitis B is not. 3. Hepatitis B is more readily transmitted via needle sticks than HIV. 4. Neither virus is transmitted via body fluids.

3. Correct: Hepatitis B virus (HBV) and HIV can be transmitted in similar ways, but hepatitis B is more infectious. Studies show hepatitis B is more readily transmitted via needle sticks than HIV. More than 1 million people currently have HIV in the United States. Hepatitis B is 50-100 times more infectious then HIV. 1. Incorrect: Neither virus is transmitted via toilet seats. Both are spread by contact with infected body fluids such as blood, semen and vaginal fluid, or from a mother to her baby during pregnancy or delivery.2. Incorrect: Both hepatitis B and HIV are transmitted via body fluids through sexual contact. Therefore, condoms should be used during sexual contact. Using a latex condom reduces the chances of hepatitis B and HIV being passed on during sex. Syringes and other injecting drug equipment should never be shared. 4. Incorrect: Both hepatitis B and HIV are transmitted via body fluids through sexual contact. Standard precautions should be implemented for both HIV and hepatitis B. The CDC recommends HBV vaccination for people who are at risk for or living with HIV, including men who have sex with men (MSM); people who inject drugs; household contacts and sex partners of people who have HBV; people with multiple sex partners; anyone with a sexually transmitted infection; people with diabetes; and health care and public safety workers who may be exposed to blood on the job.

A client is admitted with an acute episode of diverticulitis. What symptom would the nurse promptly report to the primary healthcare provider? 1. Midabdominal pain radiating to the shoulder 2. Nausea and vomiting periodically for several hours 3. Abdominal rigidity with pain in the left lower quadrant 4. Elimination pattern of constipation alternating with diarrhea

3. Correct: Pain in the lower left quadrant with abdominal rigidity indicates the client is experiencing a perforated diverticuli and is a medical emergency. Abdominal rigidity indicates either perforation or internal bleeding. Both of these symptoms are considered an "acute abdomen" and are emergencies. 1. Incorrect: Midabdominal pain radiating to the shoulder is a common s/s for a client with cholecystitis but is not a medical emergency. 2. Incorrect: Nausea and vomiting periodically for several hours is often seen with diverticulitis but is not a medical emergency. 4. Incorrect: Elimination pattern of constipation alternating with diarrhea indicates a partial bowel obstruction and may require further investigation, but this is not a medical emergency.

The nurse is assigned to bathe a client diagnosed with dementia. Which nursing intervention should the nurse implement? 1. Increase the volume of the television. 2. Finish the bath as soon as possible. 3. Clean the face and hair at the end of the bath. 4. Delegate another nurse to distract the client.

3. Correct: The client with dementia begins to be separated from reality. Gradually the client will decrease their ability to perform activities of daily living. These steps in the bathing process should be performed last because bathing the face and washing their hair may upset the client. 1. Incorrect: Because the client is having difficulty processing sensory input, the loud volume on the television will increase the anxiety of the client. The television should be turned off during the bath. 2. Incorrect: Finishing the bath as soon as possible does not address the client's inability to process the bath procedure. The client's bath should not be rushed. The nurse should proceed with the bath in a calm and controlled manner that will reduce the client's anxiety. 4. Incorrect: Options that transfer nursing responsibility to other members of the healthcare team usually are incorrect.

A client diagnosed with a right embolic stroke is admitted to the rehabilitation unit. The client is presenting with dysphagia. Which nursing intervention would the nurse implement for a client with dysphagia? 1. Flex the neck backwards 2. Request a liquid diet for the client 3. Place food on the right side of the mouth 4. Turn the client's plate around halfway through the meal

3. Correct: The client's neurological deficit will determine where to place the food in the person's mouth. The food should be placed on the right side of the mouth due to the client's left facial weakness. 1. Incorrect: The neck should be flexed forward. If the neck is flexed backward, food/liquids will have more difficulty moving through the esophagus to the stomach. The client's risk of aspiration into the trachea increases. 2. Incorrect: Liquid foods are not recommended on a dysphagia diet because liquid foods will increase the risk of aspiration. The client should eat foods that are softer in texture and thickened liquids to aid their ability to swallow. 4. Incorrect: Turning the plate is an intervention for homonymous hemianopsia. Homonymous hemianopsia is losing half of your visual field in one or both eyes. The client is exhibiting dysphagia, and is not experiencing a visual deficit. Turning the plate is not an intervention for dysphagia.

A small community has experienced a mudslide that hit a restaurant causing mass casualties. What would the nurse do first? 1. Assess the immediate area for electrical wires on the ground. 2. Attend to victim injuries as they are encountered. 3. Activate the community emergency response team. 4. Triage and tag victims according to injury.

3. Correct: With mass casualties, community response teams are needed. 1. Incorrect: This would be the second step so that further injuries are not encountered. 2. Incorrect: Triage must occur before treatment of anyone so that an accurate assessment of level of injuries can be made. With mass casualties, a color tag system is usually implemented. 4. Incorrect: This would be the third step.

What should the nurse emphasize when teaching clients how to decrease the risk of chronic obstructive pulmonary disease? Select all that apply 1. Avoid exposure to individuals with respiratory infections. 2. Increase intake of Vitamin C. 3. Eliminate exposure to second hand smoke. 4. Avoid prolonged exposure to occupational dusts and chemicals. 5. Get a yearly influenza and pneumococcal vaccination.

3., & 4. Correct: The most important environmental risk factor for COPD is cigarette smoking. Second hand smoking also contributes to COPD. Risk factors for COPD include prolonged and intense exposure to occupational dust and chemicals as well as indoor and outdoor air pollution. 1. Incorrect: Exposure to individuals with respiratory infections does not increase risk of chronic obstructive pulmonary disease. Respiratory infections may cause an acute exacerbation in a client with existing COPD. 2. Incorrect: Increasing intake of vitamin C does not decrease risk of obstructive pulmonary disease. 5. Incorrect: Clients should get the influenza vaccine annually in autumn. The pneumococcal vaccine should be administered every 5 years, rather than yearly.

The nurse is discussing frostbite prevention with a group of teenagers who participate in cold weather activities. What information should the nurse provide? Select all that apply 1. Limit alcohol intake when out in cold weather. 2. Dress in several layers of tight fitting clothing. 3. Eat well-balanced meals. 4. Synthetic clothes absorb moisture and become wet quickly. 5. Wear a wool headband over the ears. 6. Wear several pairs of socks.

3., & 5. Correct: Eat well-balanced meals and stay hydrated. Doing this even before going out in the cold will help the person stay warm. People should not diet or restrict food or fluid intake when participating in winter outdoor activities. Malnutrition and dehydration contribute to cold related illnesses and injury. Heavy woolen or windproof materials make the best headwear for cold protection. 1. Incorrect: Don't drink alcohol if planning to be outdoors in cold weather. Alcoholic beverages cause the body to lose heat faster. 2. Incorrect: Dress in several layers of loose, warm clothing. Air trapped between the layers of clothing acts as insulation against the cold. 4. Incorrect: Synthetic clothes should be worn because it moves moisture away from the body and dries quicker. Cotton clothing, especially as an undergarment, holds moisture and becomes wet quickly and contributes to the development of hypothermia and frostbite. 6. Incorrect: Wearing several pairs of socks can decrease circulation and lead to frostbite. Instead, wear socks and sock liners that fit well, wick moisture and provide insulation.

Which client diagnosis would a prescription for an intravenous infusion of 1000 mL normal saline with 20 mEq (20 mmol) potassium chloride be appropriate? 1. Major burn injury 2. Kidney disease 3. Abdominal cramping with diarrhea 4. Diabetic Ketoacidosis (DKA) 5. Hypokalemia

3., 4. & 5. Correct: Clients with abdominal cramping with diarrhea, diabetic ketoacidosis, and hypokalemia are safe to receive normal saline with potassium chloride. A primary electrolyte found in the lower GI tract is potassium. Therefore, diarrhea can result in excessive losses of potassium and associated hypokalemia can occur. When insulin is given to the client in DKA, it causes a transport of both glucose and potassium out of the blood and into the cell, resulting in hypokalemia. Finally, a client who has hypokalemia from other causes would need potassium replacement as well. 1. Incorrect: Tissue destruction from a major burn will cause release of potassium from the cell and into the blood. Thus, hyperkalemia occurs. An IV infusion with potassium will make the problem worse. 2. Incorrect: With kidney disease and the resulting diminished renal function, the client is at risk for sodium and potassium retention.

A construction worker comes into the occupational health nurse's clinic reporting chest heaviness. The nurse should assess for what additional signs and symptoms? Select all that apply 1. Headache 2. Dry, flushed skin 3. Lightheadedness 4. Dyspnea 5. Irregular pulse

3., 4. & 5. Correct: The nurse should be thinking myocardial infarction (MI)! All of these are signs of an MI. 1. Incorrect: Headaches do not commonly occur with MI. 2. Incorrect: Skin would be cool and clammy.

The nurse is preparing to administer a dose of sacubitril/valsartan 24/26 mg by mouth. The nurse has not administered this medication before and is using a drug reference to review information about the medication. Which client and drug reference information supports the nurse's decision to withhold the sacubitril/valsartan? Exhibit Select all that apply 1. Bilateral crackles noted to posterior lung fields. 2. Potassium- 4.8 mEq/L (4.8 mmol/L). 3. Currently taking trandolapril 2 mg by mouth daily. 4. Concomitant use or use within 36 hours of ACE inhibitors. 5. ACE inhibitors increase risk of angioedema. 6. Decreased Hematocrit

3., 4., & 5. Correct: This client is currently taking trandolapril, which is an ACE inhibitor. The drug reference guide specifically said, "concomitant use (of sacubitril/valsartan) or use within 36 hours of ACE inhibitors" is contraindicated because giving with ACE inhibitors can increase the risk of angioedema. 1. Incorrect: Bilateral crackles noted to posterior lung fields. 2. Incorrect: Potassium- 4.8 mEq/L (4.8 mmol/L). This is within normal limits and would not require withholding the sacubitrin/valsartan. 6. Incorrect: This client's Hematocrit is 43%, which is normal. Normal values: Adult males: 42-52% (0.42-0.52). Adult women: 37-47% (0.37-0.47). Therefore, this would not influence the administration of the sacubitril/valsartan.

What signs/symptoms would the nurse expect to assess in a client diagnosed with Guillain-Barre' Syndrome? Select all that apply 1. Opisthotonos 2. Seizures 3. Paresthesia 4. Hemiplegia 5. Hypotonia 6. Muscle aches

3., 5., & 6. Correct: Guillain-Barre' Syndrome is an acute, rapidly progressing, and potentially fatal form of polyneuritis. It is characterized by ascending, symmetric paralysis affecting the cranial and peripheral nerves. Signs and symptoms include paresthesia, hypotonia, areflexia, muscle aches, cramps, orthostatic hypotension, hypertension, bradycardia, facial flushing, facial weakness, dysphagia, and respiratory distress. 1. Incorrect: Opisthotonos is extreme arching of the back and retraction of the neck. This is seen with tetanus, not with Guillain-Barre' Syndrome. 2. Incorrect: Seizures can be associated with many neuromuscular problems but are not typical with Guillain-Barre' syndrome. Look for seizures with such problems as increasing ICP, infections of the brain, high fever, epilepsy. 4. Incorrect: Hemiplegia, paralysis on one side of the body, is not seen. There is a symmetric paralysis starting in the lower extremities and ascending through the body. In other words, weakness begins in the feet and progresses upward. The client gets better in reverse order.

An Orthodox Jewish client receives the following lunch tray. What is the nurse's priority action? Exhibit: Spaghetti and meatballs in sauce Tossed salad with vinegarette dressing Hot rolls with butter Fruit cocktail Milk - coffee 1. Nothing, since this is a healthy and acceptable lunch. 2. Ask the client to eat the acceptable food and leave the remaining food on the tray. 3. Remove the tossed salad so the client can eat the rest. 4. Call dietary to provide an acceptable meal for the client.

4. CORRECT. One Jewish religious belief contends that dairy and meat cannot be served, or eaten, at the same meal. The tray includes meatballs in the spaghetti and milk served with the meal. Nothing on this tray could be consumed by the client, and an entirely new tray must be prepared immediately. 1. INCORRECT. The "healthiness" of the lunch is not the issue. The Jewish faith contends that milk and meat can neither be eaten nor served at the same meal. Because both items are on the same tray, this meal would not be acceptable for this client. 2. INCORRECT. Because milk and meat has been placed on the same tray, the client cannot eat anything from this "contaminated' meal. Expecting the client to eat anything from this tray is not acceptable. 3. INCORRECT. The issue is not the tossed salad. The problem is that dairy (milk) has been served along with a meat product at the same meal. This is not acceptable for those of the Jewish faith, and the entire tray must be removed from the client's room.

A client awaiting discharge for a broken left tibia is to be sent to physical therapy for crutches and crutch walking. The client reports having brought a pair of crutches borrowed from a family member. What is the most appropriate action for the nurse to take now? 1. Cancel physical therapy and allow client to leave. 2. Ask client to stand with crutches to check the size. 3. Tell client insurance will not permit use of old crutches. 4. Send client with crutches to physical therapy for evaluation.

4. CORRECT. The physical therapy department is best qualified to assist a client in adjusting to the use of crutches prior to discharge. Because the client wants to use older crutches, it is even more important for a physical therapist to determine whether it is safe for the client to do so. Physical therapy can evaluate the condition of the old crutches, the client's ability to manage that equipment and to walk safely with those crutches. 1. INCORRECT. It is permissible for a client to use previously owned medical equipment. However, the stability of that equipment and the client's ability to use the equipment safely must be evaluated by physical therapy. Cancelling physical therapy would also violate the physician's orders and place the client at risk for injury upon discharge. 2. INCORRECT. While the nurse may be able to adjust the old crutches to the client's height, crutch safety and walking should be evaluated by physical therapy to be certain the previous equipment is appropriate. 3. INCORRECT. Insurance does not designate whether assistive medical devices can be reused by clients or if a new device must be purchased. It is cost effective to reuse durable medical equipment if it is appropriately suited to the client's current needs.

Which nursing intervention should the nurse include when caring for a client with Alzheimer's disease being admitted to a long term care facility? 1. Offer multiple environmental stimuli at the same time to provide distraction. 2. Encourage the client to participate in activities such as board games. 3. Restrain the client in a chair to prevent falls when sundowning occurs. 4. Involve the client in supervised walking as a routine.

4. Correct: A regular routine and physical activity help client's with Alzheimer's disease maintain abilities for a longer period of time. Physical activities promote strength, agility and balance. The client's walking should be supervised for client safety issues. 1. Incorrect: Environmental stimuli should be limited with clients with Alzheimer's Disease. The client can become agitated and/or more disoriented with an increase in environmental stimuli. 2. Incorrect: Board games would not be appropriate due to the client's cognitive and memory impairment. Board games require complex cognitive actions. 3. Incorrect: Restraints should be avoided because they increase agitation. The client may become agitated by the restriction of he restraints. Also the client may perceive the restraints as a threat.

The nurse is teaching a community education class on alternative therapies. Which alternative therapy that uses substances found in nature should the nurse include? 1. Energy therapies. 2. Mind-body interventions. 3. Body-based methods. 4. Biologically-based therapies.

4. Correct: Biologically-based therapies use substances found in nature such as herbs, foods, and vitamins.1. Incorrect: Energy therapies use energy fields. Substances found in nature are biologically-based therapies.2. Incorrect: Mind-body interventions use the mind to help affect the function of the body. Substances found in nature are biologically-based therapies.3. Incorrect: Body-based methods use movement of the body. Substances found in nature are biologically-based therapies.

Which client should the nurse recognize as being at greatest risk for the development of cancer? 1. Smoker for 30 plus years 2. Body builder taking steroids and using tanning salons 3. Newborn with multiple birth defects 4. Older individual with acquired immunodeficiency syndrome

4. Correct: Cancer has a high incidence in the immune deficiency client and in the older adult with both of these risk factors together, this one is the highest risk for cancer. 1. Incorrect: Although smoking is a known environmental carcinogen, this one risk factor alone is not the highest risk. 2. Incorrect: These are known environmental carcinogens, but do not rank as highly as aging and immune deficiency. 3. Incorrect: Birth defects are not a risk factor for cancer.

The charge nurse is making assignments for the evening shift. Which client would be an appropriate assignment for a new LPN/VN graduate? 1. A middle aged adult admitted with syncope. 2. An adolescent with skin grafts to right hand. 3. A young adult receiving IV chemotherapy. 4. An elderly adult diagnosed with diverticulitis.

4. Correct: Diverticulitis is a bowel disorder of undetermined origin, characterized by pain to the lower abdomen, along with bloating, fever and diarrhea. Treatment may include hospitalization, antibiotics, liquid diet and bedrest. Because there is usually no bleeding involved, this would be an appropriate assignment for a new LPN/VN graduate. 1. Incorrect: A new admission requires assessments that must be completed by a registered nurse. Additionally, syncope could be an indication of a serious cardiac issue; therefore, this would not be a client suitable for the LPN/VN. 2. Incorrect: Although sterile wound care is within the scope of practice for an LPN/VN, skin grafts require special assessment techniques during dressing changes, in order to determine quality of tissue perfusion and potential for rejection. This client should be assigned to a more experienced nurse. 3. Incorrect: This client will need extensive assessments of multiple body systems while receiving I.V. chemotherapy, requiring an experienced registered nurse with knowledge of both chemo drugs and different types of cancer. This client would not be appropriate for a new LPN/VN.

Which risk factor should the nurse include when planning to educate a group of women about breast cancer? 1. Menopause before the age of 50 2. Drinking one glass of wine daily 3. Multiparity 4. Early menarche

4. Correct: Early menarche before age 12 is a known risk factor for breast cancer. The increased risk of breast cancer linked to a younger age at first period is likely due, at least in part, to the amount of estrogen a woman is exposed to in her life. A higher lifetime exposure to estrogen is linked to an increase in breast cancer risk. The earlier a woman starts having periods, the longer her breast tissue is exposed to estrogens released during the menstrual cycle and the greater her lifetime exposure to estrogen. 1. Incorrect: Studies show women who go through menopause after age 50 have increased risk of breast cancer. The risk for breast cancer increases as time period between menarche and menopause increases.2. Incorrect: Small increase in risk with moderate alcohol consumption, not one glass of wine daily. Drinking low to moderate amounts of alcohol, however, may lower the risks of heart disease, high blood pressure and death. But, drinking more than one drink per day (for women) and more than two drinks per day (for men) has no health benefits and many serious health risks, including breast cancer. Alcohol can change the way a woman's body metabolizes estrogen (how estrogen works in the body). This can cause blood estrogen levels to rise. Estrogen levels are higher in women who drink alcohol than in non-drinkers. These higher estrogen levels may in turn, increase the risk of breast cancer. 3. Incorrect: Nulliparity (no pregnancies) is a known risk factor for breast cancer. Factors that increase the number of menstrual cycles also increase the risk of breast cancer, probably due to increased endogenous estrogen exposure.

A client asks the nurse, "What causes hypermagnesemia?" The nurse should explain to the client that hypermagnesemia can occur secondary to what health problem? 1. Peripheral vascular disease 2. Dehydration 3. Liver failure 4. Renal insufficiency

4. Correct: Magnesium is excreted primarily through the kidneys. When the client experiences renal insufficiency, magnesium is held. The incidence of hypermagnesemia is rare in comparison with hypomagnesemia, and it occurs secondary to renal insufficiency. 1. Incorrect: Peripheral vascular disease does not lead to hypermagnesemia 2. Incorrect: Dehydration leads to the electrolyte imbalance of hypernatremia, it does not cause hypermagnesemia. A client who has become dehydrated due to excessive urination may experience hypomagnesemia. 3. Incorrect: Liver failure does not lead to hypermagnesemia. Magnesium is regulated by GI absorption and renal excretion.

The nurse checks the results of a urinalysis performed on a client with dehydration. Which results should the nurse expect to find? 1. Increased white blood cells 2. Presence of protein 3. Presence of ketones 4. Increased specific gravity

4. Correct: Specific gravity is an indicator of hydration status and urine osmolality. In a dehydrated client, specific gravity is increased, indicating highly concentrated urine.1. Incorrect: White blood cells should not be found in the urine unless an infection is present. Dehydration does not cause white blood cells in the urine. 2. Incorrect: Protein should not be found. Presence of protein indicates renal disease. In order to have proteinuria there must be damage to the glomeruli 3. Incorrect: Ketones should not be present. They are found in clients with poorly controlled diabetes or hyperglycemia, because ketones are a by-product of fat breakdown. Fats are broken down and used for energy when glucose cannot be transported into the cells because of lack of insulin.

While preparing an information sheet for a client diagnosed with a vancomycin-resistant enterococcus (VRE) urinary tract infection (UTI), the home health nurse should include which instructions? 1. Wash hands with hot water and soap when hands are soiled. 2. Gloves are not needed in the home since contamination with VRE has already occurred. 3. Wash hands before using the bathroom and after preparing food. 4. Clean the bathroom and kitchen with warm water and bleach.

4. Correct: The bathroom and kitchen should be cleaned with warm water and bleach to decrease contamination. The client should wash hands after using the bathroom and before preparing food. 1.Incorrect: Instructing the client and family to wash with hot water can cause drying and cracking of the skin. Hands should be washed with all contacts. Washing hands is the single most important thing to do to prevent infection. 2. Incorrect: Gloves are needed with VRE to prevent spread of infection. Gloves are especially needed if contact with blood or other infectious materials is anticipated. 3. Incorrect: Hands should be washed after using the bathroom and prior to handling or preparing food.

The nurse is caring for a client who has been intubated and placed on a ventilator. The nurse hears the ventilator alarm and enters the client's room to find the high pressure alarm sounding. The client is very agitated with a respiratory rate of 40/min, arterial BP 98/48, oxygen saturation 82%, cardiac monitor showing sinus tachycardia at 138 bpm. What action should the nurse take first? 1. Turn off alarm, then check ventilator settings. 2. Increase FiO2 settings to 100%. 3. Hyperventilate client. 4. Auscultate lung sounds.

4. Correct: When an alarm sounds, the first action by the nurse is to assess the client. In this situation, assessment of lung sounds, chest movement, and respiratory effort should indicate which respiratory complication the client may be experiencing. Depending on the assessment findings, the other actions may be necessary. 1. Incorrect: Depending on the assessment findings, the other actions may be necessary. Check the ventilator after checking the client. 2. Incorrect: Depending on the assessment findings, the other actions may be necessary. 3. Incorrect: Depending on the assessment findings, the other actions may be necessary.

The nurse is caring for a post op client who is drowsy but arousable. The client will take a few deep breaths when instructed but drifts to sleep when left alone. The O2 saturation while sleeping drops to 82% on 3 liters of nasal oxygen. The client received a dose of oxycodone/acetaminophen 2 tabs one hour ago. What is the nurse's best action at this time? 1. Keep the O2 sat machine at the bedside and set the alarm to beep loudly when O2 sat drops below 93%. 2. Give bath to arouse client and then report that oxycodone/acetaminophen 2 tabs is too much for next dose. 3. Let the client sleep until he has rested, then discuss abuse potential of narcotics. 4. Call the primary healthcare provider and report client assessment findings.

4. Correct: Yes, this client has unstable respirations and is in respiratory distress. The client needs naloxone,the antidote for narcotic overdose. Since that is not an option, you need to call the primary healthcare provider to get a prescription for the antidote. 1. Incorrect: That will work the first time, but the client is too sedated to remain awake and take deep breaths. The client will continue to have respiratory distress until naloxone can be given. 2. Incorrect: No, that won't fix the problem of too much medication. We need to fix the problem now. 3. Incorrect: No, client is too sedated. Naloxone is needed, so the nurse needs to notify the primary healthcare provider.

What assessment finding would alert the nurse that a client's open pneumothorax has progressed to a tension pneumothorax? Select all that apply 1. Subcutaneous emphysema 2. Shortness of breath 3. Tachypnea 4. Distended neck veins 5. Hypotension

4., & 5. Correct: As pressure builds inside the chest cavity, a tension pneumothorax can develop. Symptoms that are commonly seen with a tension pneumothorax include distended neck veins, distant heart sounds, and hypotension. Now, why does the tension pneumothorax client experience distended neck veins? Because the pressure is building up in the chest causing pressure on the right side of the heart. This is a bad thing! There is decreased venous return and this can lead to cardiovascular collapse. Because of the compromised or obstructive blood flow, cardiac output decreases and hypotension develops. 1. Incorrect: You would expect to see subq emphysema with an open pneumothorax and can still be seen with a tension pneumothorax. 2. Incorrect: SOB is expected with open pneumothorax and tension pneumothorax. 3. Incorrect: Tachypnea would be seen with both.

The nurse is caring for a client who sustained a head injury with possible seizure activity. The primary healthcare provider prescribes an EEG. What client teaching should the nurse provide to the client prior to this test? 1. Instruct client to be NPO after midnight. 2. Tell client not to wash their hair the night before the test. 3. Assure client that they may take sleeping pill the night prior to the EEG. 4. Instruct client not to drink caffeinated beverages the morning of the test. 5. Take routine medications the morning of the EEG with a sip of water.

4., & 5. Correct: Caffeine is a stimulant and will speed up the electricity in the brain. We don't want to do anything that is going to alter a client's electrical activity in the brain before an EEG. The cliebts are encouraged to take their routine medications unless directed otherwise. 1. Incorrect: Fasting may cause hypoglycemia and could alter the EEG pattern. 2. Incorrect: The client should wash hair the night before the test. 3. Incorrect: No sedatives or hypnotics are given before the test. This could cause abnormal results on the EEG. In addition, the client may be directed to sleep less the night before the EEG if sleeping is desired during the EEG.

A 35 year old client, concerned about weight, asks a clinic nurse, "What is my BMI?" The client weighs 135 pounds and is 5 feet 2 inches tall. Determine the client's BMI to the nearest tenth?

BMI = 24.7 Formula: BMI = (703 x weight in pounds) ÷ (height in inches)2 BMI = (703 x 135) ÷ (62)2 -62x62=3,844 BMI = (94,905) ÷ (3,844) BMI = 24.689 BMI = 24.7

The nurse is demonstrating ostomy care to a client with a new stoma in the sigmoid area of the colon. The nurse knows teaching is successful when the client completes care in what order? Cut center of new flange to fit stoma. Remove ostomy bag and old flange. Apply skin protectant and allow drying. Press flange into place and attach bag. Wash stoma with warm soapy water. Place stoma adhesive onto new flange.

Changing an ostomy appliance can be challenging for an individual without assistance from another individual. Successful completion of the procedure involves not only a willingness to learn, but also the client's physical capabilities to reach, view, and accomplish the task based on the type or location of stoma. The basic principles of stoma care are similar to any dressing change. The client must remove and dispose of the old, inspect and clean the area, then prepare and apply the new appliance. Though there are multiple steps, many clients can perform self-ostomy care with practice and minimal assistance. The ostomy flange is designed to remain in place for 3 to 5 days, while the bag can be changed, or cleaned, daily. There are many types of appliances, including some which are sealed and are simply thrown away at the end of each day. The steps have been simplified here, but are basic: First, remove both the ostomy bag and old flange. Second, thoroughly wash the entire abdomen with warm soapy water, being sure to wash stoma gently. (The client must be instructed to carefully inspect the skin for any signs of excoriation which should be reported to the primary healthcare provider). Third, a skin protectant is applied and allowed to dry. Fourth, the new flange is sized/cut to fit the stoma. Fifth, a thin "bead" of stoma paste is placed around the new flange opening. Lastly, Press flange into place and attach the bag. The bag is snapped over the center rim of the flange, and "burped" to allow any excess air to escape. This will create a strong seal. It is also vital to apply the clamp onto the bottom of the new bag to prevent leakage of stool.

The nurse is caring for a client receiving peritoneal dialysis. Place the steps for peritoneal dialysis in the correct order. Drag and Drop the items from one box to the other Warm dialysate Access Tenckhoff catheter Begin dwell time Complete exchange Assess effluent

First, warm the dialysate. Would you put cold or even cool dialysate in your peritoneal cavity? NO, it would feel uncomfortable but more importantly, it would vasoconstrict the vessels of the peritoneal membrane. Would that affect the success of the dialysis? Yes, you want dilated blood vessels to promote osmosis and diffusion. Second, access the Tenckhoff catheter, assess it for patency and look at the site for infection. Just like you would do with any IV access. Third, begin the dwell time. Fourth, complete exchange by removing effluent by gravity drainage. Fifth, assess effluent. What would I assess the effluent for? Color, clarity, amount...just like urine? What am I worried about? Infection. How would my effluent look if I had an infection? Cloudy. If I have an infection in my peritoneal cavity, I need immediate antibiotic therapy to prevent peritonitis and damage to the membrane. The primary healthcare provider is going to want to culture the effluent and start a broad spectrum antibiotic.

The nurse initiates sterile wound care on a client's newly debrided foot ulcer. After removing the dressing and beginning a betadine cleanse, the client mentions an allergy to iodine not previously reported. Place the nursing actions in order of priority. Observe client for signs or symptoms of reaction. Remove betadine solution from wound with normal saline. Cover wound with temporary sterile dressing. Ask client about the type of "allergic response". Notify primary healthcare provider of the allergy.

First: Because many individuals confuse the term "allergy" with expected side effects, the nurse first needs to quickly determine the type of response this client may have previously experienced. Second: Whether or not the client is able to give a clear description of an allergic response, such as hives, swelling or reddened skin, the nurse must assume the worst and then immediately remove the betadine solution from the open wound/skin before any reaction occurs. Third: The previously unreported allergy may require a change in the plan of care; however, at this moment the nurse has a fresh wound exposed to air. The nurse should cover the wound with a dry sterile dressing. Fourth: Then observe the client for at least 10 minutes to determine the need for any emergency intervention in case of anaphylaxis. Fifth: Once it is determined the client is stable, the healthcare provider should be contacted regarding the new information, client's status and whether new wound care orders are needed.


Ensembles d'études connexes

mod 4 data collection, behavior/ decisions

View Set

Med Surg Ch 47 Intestinal & Rectal Disorders

View Set

EVS lecture 6 (chapter 7: waste), lecture 7 (chapter 10: air pollution), & lecture 8 (chapter 11: ozone depletion)

View Set

CONCEPTOS RELACIONADOS CON LA SEXUALIDAD

View Set

Learning curve 3a, b, c: Developmental Psychology's Major Issues; Prenatal Development and The Newborn

View Set

ECON 101: Principles of Microeconomics Final Exam Practice

View Set